Jump to content

Wikipedia:Reference desk/Science: Difference between revisions

From Wikipedia, the free encyclopedia
Content deleted Content added
No edit summary
Line 317: Line 317:
::It would also depend on the depth of the snow and how long it has been in place. Right after an avalanche, the surface of the snow should be about the same temp everywhere, while, after some time, the snow near the body should be slightly warmer. [[User:StuRat|StuRat]] ([[User talk:StuRat|talk]]) 04:19, 31 July 2011 (UTC)
::It would also depend on the depth of the snow and how long it has been in place. Right after an avalanche, the surface of the snow should be about the same temp everywhere, while, after some time, the snow near the body should be slightly warmer. [[User:StuRat|StuRat]] ([[User talk:StuRat|talk]]) 04:19, 31 July 2011 (UTC)
:::Unfortunately, by that time most of the victims would be dead. See [[Avalanche#Human survival and avalanche rescue]]. [[Special:Contributions/67.169.177.176|67.169.177.176]] ([[User talk:67.169.177.176|talk]]) 05:19, 31 July 2011 (UTC)
:::Unfortunately, by that time most of the victims would be dead. See [[Avalanche#Human survival and avalanche rescue]]. [[Special:Contributions/67.169.177.176|67.169.177.176]] ([[User talk:67.169.177.176|talk]]) 05:19, 31 July 2011 (UTC)

==<del>fart. Erm,</del> flatulence==
<del>Exactly what is a fart composed of? Oh, that's obvious, it depends on what you eat/drink/do/say/whatever, but w</del> What is the general base for a fart? Is it true that one can fart next to a lighter and get the fart fire cloud thingy? '''''[[User:Since 10.28.2010|<font color="#DAA520">An</font>]] [[User talk:Since 10.28.2010|<font color="#DAA520">editor since 10.28.2010.</font>]]''''' 05:37, 31 July 2011 (UTC)

Revision as of 05:37, 31 July 2011

Welcome to the science section
of the Wikipedia reference desk.
Select a section:
Want a faster answer?

Main page: Help searching Wikipedia

   

How can I get my question answered?

  • Select the section of the desk that best fits the general topic of your question (see the navigation column to the right).
  • Post your question to only one section, providing a short header that gives the topic of your question.
  • Type '~~~~' (that is, four tilde characters) at the end – this signs and dates your contribution so we know who wrote what and when.
  • Don't post personal contact information – it will be removed. Any answers will be provided here.
  • Please be as specific as possible, and include all relevant context – the usefulness of answers may depend on the context.
  • Note:
    • We don't answer (and may remove) questions that require medical diagnosis or legal advice.
    • We don't answer requests for opinions, predictions or debate.
    • We don't do your homework for you, though we'll help you past the stuck point.
    • We don't conduct original research or provide a free source of ideas, but we'll help you find information you need.



How do I answer a question?

Main page: Wikipedia:Reference desk/Guidelines

  • The best answers address the question directly, and back up facts with wikilinks and links to sources. Do not edit others' comments and do not give any medical or legal advice.
See also:


July 27

Motor type and most likely wiring scheme

I was given the motor to the right.

Motor

It was taken from an old window AC unit in the US. I think it's a 3-Phase motor but all the 3-phase_AC color codes both inside the US and outside that I've seen have other than what this motor has: black, brown, red, blue, and gray. Peter Michner (talk) 01:26, 27 July 2011 (UTC)[reply]

There is nothing recognisable in the photograph to help gauge the size of the motor. (Also the photograph appears to be upside down, or is it attached to the ceiling?) I doubt a smallish motor for a window air conditioner would be 3-phase. Perhaps it is a single-phase motor and the extra cables are to facilitate some operating variation such as two-speed or forward-reverse. Dolphin (t) 03:37, 27 July

2011 (UTC)

Sorry about that. It's about 10cm in diameter, (and sitting on the counter in that photo) so about the size of a regular person's fist. The two other pieces of information it gives are a GE sticker on one side and another sticker that says "HAM-3500(W) 120V 60Hz." Peter Michner (talk) 15:03, 27 July 2011 (UTC)[reply]
If you Google Images "air conditioning stepper motor" you get pictures of similar motors with the same number of wires (5). Therefore I think it's a stepper motor, probably used to control the vent direction. This means you need to connect it to a stepper motor driver, not direct to the mains. --Heron (talk) 08:30, 27 July 2011 (UTC)[reply]
Interesting. I'd like to see the exact pictures you were looking at, because all the stepper motor wiring diagrams I could find had even numbers of wires, like four or six. Peter Michner (talk) 15:03, 27 July 2011 (UTC)[reply]
Here is picture #1 in my search results. It has 5 wires. However, from the other information given in this thread after I replied, it seems that the motor in question is not a stepper. --Heron (talk) 17:12, 27 July 2011 (UTC)[reply]
If its a stepper motor, you may be able to feel 'cogging' as you rotate the shaft by hand.--GearCutter (talk) 12:55, 27 July 2011 (UTC)[reply]
Actually, it's perfectly smooth. Peter Michner (talk) 15:03, 27 July 2011 (UTC)[reply]
I just took an ohmmeter to the wires and came up with this table:
Color Black Brown Blue Gray Red
Black X 25.0 Ω 5.1 Ω ∞ Ω 38.9 Ω
Brown 25.0 Ω X 21.9 Ω ∞ Ω 42.0 Ω
Blue 5.1 Ω 21.9 Ω X ∞ Ω 39.9 Ω
Gray ∞ Ω ∞ Ω ∞ Ω X ∞ Ω
Red 38.9 Ω 42.0 Ω 39.9 Ω ∞ Ω X

|} This would indicate gray was ground, right? Peter Michner (talk) 22:43, 30 July 2011 (UTC)[reply]

That would be a very odd color for a ground lead. What is Ω for Gray to frame? If the roation is perfectly smooth you probably have some type of AC induction motor and the "extra" leads are for speed control or cap start or the like. I'd suggest walking into a local repair shop, preferably one that re-winds on site, and asking. - ArtifexMayhem (talk) 03:06, 31 July 2011 (UTC)[reply]
Infinity there, too. It's as though the gray wire isn't connected to anything. I opened the motor to see a solid squirrel cage inside, so it's definitely an induction motor. I'll try to find a motor repair shop in my area, probably. Peter Michner (talk) 23:31, 31 July 2011 (UTC)[reply]

Identify butterfly or moth species

The species in question.
View #2.

What species of butterfly or moth (I assume it's one of the two) is this? The photos were taken in Hot Springs National Park near Hot Springs, Arkansas in the United States. Thanks in advance, Ks0stm (TCG) 02:26, 27 July 2011 (UTC)[reply]

It's definitely a butterfly. Butterflies hold their wings vertically at rest, moths fold theirs flat. I think it's a type of swallowtail butterfly. SemanticMantis (talk) 03:54, 27 July 2011 (UTC)[reply]
Your image agrees closely with this one [1], which says it is a spicebush swallowtail. Our article spicebush swallowtail has a similar but different picture. The differences could be due sexual dimorphism, or perhaps more than one species goes by that common name. SemanticMantis (talk) 04:03, 27 July 2011 (UTC)[reply]
I'm doubtful of this assignment - a spicebush swallowtail is a swallowtail, after all, but I can't see the slightest trace of the tail on the wing after which it is named. And mimicry is common among butterflies... Wnt (talk) 06:17, 27 July 2011 (UTC)[reply]
To begin with, I see only two legs in the side view photo, so I'm thinking Nymphalidae, not Papilionidae. Wnt (talk) 06:24, 27 July 2011 (UTC)[reply]
This looks like Limenitis, maybe Limenitis arthemis astyanax, which is native to that region. Wnt (talk) 06:32, 27 July 2011 (UTC)[reply]
Yep, I think you've got it with astyanax. (`Four-footed' does rule out Papilionidae, but many swallowtails have no visible swallowtail, no?). SemanticMantis (talk) 13:35, 27 July 2011 (UTC)[reply]
Sorry for the confusion - you're right, it's just that the spicebush swallowtail does have one. Wnt (talk) 18:05, 27 July 2011 (UTC)[reply]

As seen in Knowing (film), could a solar-megaflare kill literally everyone on Earth?

When I saw the spoilers of the end-scenes on YouTube, my first thoughts were:

  1. What about anyone in bunkers deep underground? Could they survive there?
  2. How about in submarines deep under the oceans? How far down could the solar flares hit there?

I would have confidence that being deep enough underground or underwater would ensure survival. What do you make of this? --70.179.165.67 (talk) 05:55, 27 July 2011 (UTC)[reply]

There's no definition of how big a flare has to be before you call it a 'megaflare'. It could just mean 'bigger than the last biggest one'. Presumably if the sun decided to throw half of its mass in our direction then we would be fried. However, in the real world, even the most pessimistic scientists say that a huge solar flare would do nothing more than blow up our power grids.[2] That would be bad enough--millions in the developed world would die--but we would eventually repair all the damage and carry on. I've never heard a serious prediction of a solar flare that could set directly fire to things on the Earth's surface, or even give you sunburn. --Heron (talk) 08:24, 27 July 2011 (UTC)[reply]
I just (literally minutes ago) finished reading Sunstorm (novel), which is all about the possibility solar burst wiping out the planet, going kilometres deep and boiling off the ocean. (I recommend it by the way)--Jac16888 Talk 12:20, 27 July 2011 (UTC)[reply]
"Millions in the developed world would die." Wrong. People dependent on life support in hospitals would die, yes, as would (possibly) some people who were using technology to survive a massive heat wave or a terrible storm, but as the flare would not knock out the combustibility of objects, people would be able to create fires and stuff to survive the winter, and sprinkle water on themselves to survive the summer heat, at least until they got the power grids back on-line. Whoop whoop pull up Bitching Betty | Averted crashes 23:22, 27 July 2011 (UTC)[reply]
I think that A sudden destruction of our electrical and telecommunications infrastructure would be a lot more devastating than you believe it would. For example, the sudden shortage of refrigeration would cause food shortages. Fuel shortages would be a serious problem. Partially because people would be burning gas in generators, partially because almost every step of the process of getting oil from the ground into our cars relies on electric pumps. The economy would suffer greatly, that alone could cause huge number of deaths.
After a week or so, safe fresh water would be difficult to come by in many areas. Even in areas with plenty of water electricity is often required to pump it into water towers.
Virtually our entire economy is dependent on "Just in time" deliveries and manufacturing. Without telecommunications and unlimited gasoline that wouldn't work properly, so even if there was enough food, it wouldn't be in the stores that need it. If those trucks stop rolling and those ships stop sailing people will starve.
Even easy things like staying warm in the winter. You talk about building fires, but where are you going to get enough wood to warm up all ten million inhabitants of New York City alone? And then how are you going to get it to where it needs to be? I'm not sure we could harvest and move that much wood now, let alone in the middle of a global crisis.
After the power was turned back on there would be long term effects. If this happened during the growing or harvesting season, vast amount of crops might be lost. Which could lead to serious starvation. (Remember, we won't be able to just buy food from other countries if they all had the same problem!)
And this all assumes that electronics aren't also damaged in the flare. If microchips are knocked out as well, the vast majority of cars and trucks won't start even if they do have gas! That will make it even harder to transport supplies. (Including gasoline, and the parts we'd need to fix things! Which just compounds the problems!)
It wouldn't be the end of the world, it could all be fixed in time. But it would be far worse than you're imagining, because you're only thinking of consumer uses of electricity. You don't realize that everything else you need to stay alive also depends on electricity. We simply can't go back to an agrarian non-technological life-style with this many people, we need our technology.
APL (talk) 00:09, 28 July 2011 (UTC)[reply]
You grossly underestimate the importance of electricity and the cascading failures such as meltdowns and dam failures that would occur, not to mention huge crop losses, deterioration of frozen foods and medicines, and fires in cities, if there were a loss of generators and transformers. Just as a complete continent wide EMP failure would likely kill millions. A world-wide event would kill hundreds of millions easily.
See the FEMA report. μηδείς (talk) 00:21, 28 July 2011 (UTC)[reply]
A dam is not held up by electricity. Whoop whoop pull up Bitching Betty | Averted crashes 02:03, 31 July 2011 (UTC)[reply]
Don't be juvenile. Of course they're not held up by electricity, but they're not just simple walls in the water either. (Unless we're talking about the ones made by beavers.) Many of them have moving parts to regulate how much water is allowed through the dam. It's not difficult to imagine that if those were damaged in the flare, or could not be powered, there could be some serious results. (Letting too much or too little water through could lead to flooding on one side and water shortages on the other. And letting too little water through until the built up water destroys the dam would be a catastrophe.) APL (talk) 04:00, 31 July 2011 (UTC)[reply]
Don't they have backup power? I've been to the Hoover Dam, and sure enough it has two Pelton wheels for backup power, in addition to the main turbines. Maybe the ones in Europe don't, but American ones sure do... -- An American ultranationalist 67.169.177.176 (talk) 04:35, 31 July 2011 (UTC)[reply]
Not related to the original question, but does Knowing remind anyone else of Heinlen's The Year of the Jackpot? --Trovatore (talk) 00:24, 28 July 2011 (UTC)[reply]
A "mega-flare" of this type could only conceivably occur when the Sun begins to shed its outer layers.
Or when its luminosity begins to exceed the Eddington limit, whichever comes first. Whoop whoop pull up Bitching Betty | Averted crashes 02:03, 31 July 2011 (UTC)[reply]

DNA question

What is the difference between:

  1. my DNA and my brother's DNA
  2. my (ans my brother's) DNA and a chimp's DNA

I just don't know what kind of differences they are. What is in common in the DNA of all members of a species, and what is different in every member? when they say that 99 percent of human and chimp DNA is the same,what are they exactly talking about?Do I have to study genetics in university in order to fully understand it?--Irrational number (talk) 07:31, 27 July 2011 (UTC)[reply]

This definitely won't be the full answer you're looking for, but consider the vast number of boring mundane operations that need regulating in the body and the make up of tissue and cells. Most of the differences between humans and chimps are superficial, and between humans and cabbages for that matter. Hope this helps. Pascal (talk) 07:58, 27 July 2011 (UTC)[reply]
If you're looking for raw numbers, humans and chimpanzees share 96% of their DNA, while the average humans differ by only 0.1% (meaning they share 99.9% of their DNA). You can find more information at Human genetic variation. -RunningOnBrains(talk) 09:57, 27 July 2011 (UTC)[reply]
That 0.1% turns out to be 4 megabytes worth of individual human genetic variation, per Human genome#Information content. 99.17.204.52 (talk) 10:20, 28 July 2011 (UTC)[reply]
DNA is basically just a (really, really, really) long code of molecules. These codes govern basically everything about your body: making sure your legs grow in the right way, making sure you can digest food properly, etc. A lot of the basic structure in life is similar, throughout ALL life, so a good percentage of these codes are the same in many different species of animals (and even plants!). The more similar the lifeforms, the higher percentage DNA will be the same, which is why chimps, which are primates like us, have 96% similar DNA. But even in other mammals, like squirrels or yaks or beavers, you will find a good chunk of their DNA is similar to ours (don't have the exact numbers, but yeah). Lizards would have a smaller (but still high) percentage, fish probably lower. But yeah, two humans are pretty much the same, DNA-wise. It's tiny, tiny differences in the DNA code that make every person unique (except for identical twins, which have 100% similiar DNA).--66.207.206.210 (talk) 14:21, 27 July 2011 (UTC)[reply]
For more perspective, we share 70% of our DNA with sea sponges. -RunningOnBrains(talk) 14:58, 27 July 2011 (UTC)[reply]
Is it known what percentage of human DNA would also be found in an amoeba? Or in E coli? Has someone drawn up an extended family tree? Wanderer57 (talk) 15:27, 27 July 2011 (UTC)[reply]
That question raises a difficulty that is important to understand. Most of the exons (the parts of the DNA that encode protein structures) in human DNA correspond to exons in the DNA of amoeba and bacteria, and it is possible to compare them -- that's how the numbers that you see are derived. However, less than 1% of the total DNA belongs to exons. For the remaining DNA (99% of the total), there is no well-established method of doing cross-species comparisons. For a long time most biologists thought that the "non-coding" DNA is mainly junk, but over the last decade it has become clear that there is a lot of important stuff there. Looie496 (talk) 16:07, 27 July 2011 (UTC)[reply]
Here is the basic thing you have to understand. A DNA molecule is a string of "nucleotides". There are four possible nucleotides, usually labeled A, G, C, and T. So a DNA molecule can be thought of as a string like "AGGACTTACCTAGGACATTTG...". The string for each DNA molecule is around a hundred million letters long, though. If you compare the DNA molecules in your cells with those in your brother's cells, they can be matched up letter for letter, but there will be a few differences -- something less than 1% of the letters for your DNA will differ for your brother. For somebody less related to you, the number of differences will be larger, and for a different species, larger yet. Looie496 (talk) 15:59, 27 July 2011 (UTC)[reply]
Thanks very much. So of the 1% or so of human DNA that encodes protein structures, is it known how much would also be in an amoeba, a bacterium, a nematode, an aphid, a tobacco mosaic virus,...? Is there a lookup table? Wanderer57 (talk) 16:59, 27 July 2011 (UTC)[reply]

So is there a accurate boundary between species with regard to DNA difference(or at all)? for example, is it right to say "if the difference is more than X%, then the living things are not of the same species"?--Irrational number (talk) 18:58, 27 July 2011 (UTC)[reply]

No, there are no official boundaries. In large part, this reflects the fact that much taxonomic nomenclature predates the discovery of DNA. For modern practice though, there are some rules of thumb. With bacteria (where reproductive definitions of species are moot), typically one requires a new strain differ by at least 1% from existing species before a new species name is assigned. Further, one generally expects at least a 4% difference from all existing genera type specimens before proposing a new genera name. Dragons flight (talk) 20:14, 27 July 2011 (UTC)[reply]

Provided you have the same mother, you probably share the same mtDNA.Smallman12q (talk) 02:52, 28 July 2011 (UTC)[reply]

I was going to look this up but never got around to it, and won't for a while now - but in general, viruses are extremely efficient in using DNA to code proteins - actually going over 100% sometimes, because more than one gene is coded in the same space due to frameshifts. Bacteria spend a short stretch here and there on regulation rather than protein coding, but do so on a tight budget. The others get increasingly inefficient as their replication rate decreases, and I'd really have to look them up, because it can vary WIDELY among closely related organisms. For example there is a fish Fugu rubripes which has a very compact genome, even though other fishes have all kinds of spam shoehorned in between the genes. And of course humans spend DNA without a thought, it's like some kind of Usenet feed full of Make.Money.Fast and Nigerian bank scams. Wnt (talk) 15:09, 28 July 2011 (UTC)[reply]

Starting up a nuclear reactor

Where does the neutron used to start a nuclear reactor come from? Whoop whoop pull up Bitching Betty | Averted crashes 15:07, 27 July 2011 (UTC)[reply]

Spontaneous fissions. Nuclear bombs, on the other hand, don't work properly with such a slow-cooking method of starting up (they "energetically dismantle" themselves before they've really gotten going); so they use a neutron source like a modulated neutron initiator. -- Finlay McWalterTalk 15:12, 27 July 2011 (UTC)[reply]
The joy of a critical mass is that you don't really need to start it with anything special — once you have the material into a critical configuration, any stray neutrons will start the process. There are lots of neutrons out and about in the world, just zipping along, and as Findlay points out, your fissile material (much less impurities hanging out in the fissile material, like Pu-240) already have background neutron rates due to spontaneous fission. With a reactor you have time to just wait for the thing to start up and grow slowly; as Findlay points out, you can't do that with a bomb, because you need all of those reactions to take place in less than a millisecond. --Mr.98 (talk) 15:25, 27 July 2011 (UTC)[reply]
To show how easy it is to start a nuclear reactor up, look at Chicago Pile 1, the first nuclear reactor, and see how primitive it was. Most of the complexity in modern nuclear reactors isn't in getting them started, it is in slowing them down and stopping them. --Jayron32 16:22, 27 July 2011 (UTC)[reply]
Speaking of which, the RBMK style of reactor uses natural uranium, light water, and graphite. I wonder whether - in theory - it would have been possible (survival optional) for people in Neolithic times to create a nuclear reactor using only a natural uranium ore mixed in with soot at the right concentration... Wnt (talk) 18:02, 27 July 2011 (UTC)[reply]
In theory, probably yes, after all, it has happened naturally without any human intervention at all. In practice, of course, Neolithic peoples would have lacked the very detailed knowledge necessary. Still - intriguing plot idea. {The poster formerly known as 87.81.230.195} 90.197.66.97 (talk) 19:19, 27 July 2011 (UTC)[reply]
That happened about 2 billion years ago when the natural enrichment of uranium was about 4% of U-235. Nowadays that enrichment is only about 0.7% U-235 which wouldn't be enough for a natural reactor to occur. That's why the difficult step of enriching the natural uranium must be taken before it can be used as fuel for a nuclear reactor. Dauto (talk) 20:27, 27 July 2011 (UTC)[reply]
If that were true it would be impossible to operate a RBMK reactor with light water and natural uranium. However, it is possible, so it's possible, in theory, that Neolithic peoples could have created an (albeit very simple) nuclear reactor. However, as it would have gotten hot enough to boil away the water used to cool it, it would probably go supercritical, melt, or both, meaning that it would probably be useless for anything other than starting fires. OTOH, though, it would have been darn near impossible to shut it down, so it could have been very useful as an area denial weapon or to destroy the land of an opposing tribe or, if dropped into water, as a sort of artificial hot spring which could have been used for military (to gather buckets of scalding hot water to use as weapons, and to wash weapons more efficiently,) civilian (to wash clothing and stuff more efficiently,) punishment (to scald people to death as a form of capital punishment,) or culinary (to boil fish,) purposes. Whoop whoop pull up Bitching Betty | Averted crashes 23:33, 27 July 2011 (UTC)[reply]
That's true. But that would be a very tall order for neolithic people. My point was that the naturally occurring reactors happened during a time when it would have been much easier to produce a sustained nuclear reaction even without graphite as a moderator. Dauto (talk) 00:01, 28 July 2011 (UTC)[reply]
To be clear, I really don't know if any uranium ore is free enough from the wrong kinds of impurities to be able to start a chain reaction, nor whether enough would occur in one place that it is plausible for someone at a low tech level to collect enough of it. But it seems an amusing scenario for fiction. Wnt (talk) 15:01, 28 July 2011 (UTC)[reply]
No, a Neolithic tribe couldn't have made a nuclear reactor. Light water is trivial to come by, and smelting uranium might be possible, but there's no way to get the hyper-pure graphite needed -- a minor, unrecognized contamination of their graphite is one of the reasons the German atom bomb project failed. --Carnildo (talk) 01:46, 29 July 2011 (UTC)[reply]
Smelting is how sulfides are reduced to the bare metal. With uranium, you need to reduce the oxide to the bare metal—you would need something more along the lines of a blast furnace. Whoop whoop pull up Bitching Betty | Averted crashes 02:14, 31 July 2011 (UTC)[reply]
Didn't they have something of the sort in ancient Rome? 67.169.177.176 (talk) 04:49, 31 July 2011 (UTC)[reply]
Hmmm, nuclear graphite didn't explain this much, though I'd wondered... still, I'm surprised there's enough boron to foul up the reaction (I assume) everywhere soot can be manufactured. Wnt (talk) 04:05, 29 July 2011 (UTC)[reply]
See Nuclear reactor physics#Starter sources and Startup neutron source. Note that besides providing a reliable source for startup neutrons, they also provide an operational check of the low level power meters before startup. -- 203.82.81.81 (talk) 23:49, 27 July 2011 (UTC)[reply]
See also neutronium. ~AH1 (discuss!) 23:35, 28 July 2011 (UTC)[reply]
Um, why? It looks pretty irrelevant to me. --Mr.98 (talk) 02:03, 29 July 2011 (UTC)[reply]
The reason the Germans failed is because they didn't have the Borrowdale graphite. Conceivably they (a Neolithic tribe) could have accidentally placed a lump of uranium (wow, awesome dense armor metal) next to a (couple of) lump(s) of Borrowdale graphite (wow, awesome fuel and marking material) and hey presto, the first nuclear reactor. Whoop whoop pull up Bitching Betty | Averted crashes 02:14, 31 July 2011 (UTC)[reply]
Except that you need a lot more uranium than that to have a reactor. CP-1 is probably about as crude a uranium-graphite reactor that you can make (almost by definition, given that it's basically one critical mass, I think), and it required several tons of uranium in a lattice. It isn't the sort of thing you are going to create accidentally. A more plausible (but still silly) idea is slightly less neolithic (but still quite early) people saying, "hey, this graphite would make a great brick! but I bet it'd be better if it had a lump of this dense metal in it! we can make great houses from this! but first let's just pile them all up over here in a big heap..." But it's still a very silly scenario (it would require smelting the uranium and making it into metal, which was not the most straightforward process even for 20th century metallurgists; I don't think uranium is ever found as a metal in the wild). --Mr.98 (talk) 02:49, 31 July 2011 (UTC)[reply]
Not—smelting! Blast—furnace—ing! Whoop whoop pull up Bitching Betty | Averted crashes 03:02, 31 July 2011 (UTC)[reply]
Hmm... does tungsten carbide (excellent neutron reflector and critical—mass—reducer) exist in the wild? Whoop whoop pull up Bitching Betty | Averted crashes 03:05, 31 July 2011 (UTC)[reply]
And are there any natural processes which can enrich uranium? Whoop whoop pull up Bitching Betty | Averted crashes 03:07, 31 July 2011 (UTC)[reply]
At Oklo, the uranium was (supposedly) enriched by groundwater leaching. FWIW -- An American ultranationalist 67.169.177.176 (talk) 04:46, 31 July 2011 (UTC)[reply]
So why couldn't the Neolithes have come across such naturally-enriched uranium (wow! great armor metal!) and just happened to place it within blocks of good Borrowdale graphite and thereby create the first (primitive) nuclear reactor? Whoop whoop pull up Bitching Betty | Averted crashes 16:03, 31 July 2011 (UTC)[reply]
What? At Oklo the uranium was enriched because uranium was more highly enriched 2 billion years ago than it is today. Not because it was naturally enriched in some way. The only thing "enriched" by groundwater leaching at Oklo, I am fairly sure, was the concentration of uranium ore as a whole (not the concentration of U-235 relative to U-238) into an amount that could be of sufficient size for a critical mass. --Mr.98 (talk) 20:37, 31 July 2011 (UTC)[reply]
Go eat uranium-flavored Sagittaria! Whoop whoop pull up Bitching Betty | Averted crashes 21:13, 31 July 2011 (UTC)[reply]
Why would tons of natural uranium be needed? Critical mass is reportedly much less, and RBMK says a big reactor contains 114 kilograms. Wnt (talk) 22:58, 31 July 2011 (UTC)[reply]

Only one higgs?

On the page List of unsolved problems in physics, the section on the Higgs mechanism says: "Does the Higgs particle exist? What are the implications if it does not? Is there only one of them?"
Only one? What? The Higgs boson article says nothing about there possibly being only one. Can someone go into this a bit for me, why we think there might be only one, and what the implications are if that is the case. Thanks. --66.207.206.210 (talk) 15:44, 27 July 2011 (UTC)[reply]

Just to be sure we're on the same page, I read the statement as saying that there is only one type of Higgs particle. Do you read it as saying there is only one particle? Looie496 (talk) 15:49, 27 July 2011 (UTC)[reply]
(ec) The standard model has only one higgs particle. That's the simplest possible assumption, but nature could be more complex than that and there could be in principle many different higgs particles. Supersymmetric models usually require at least four different higgses one of which is a charged particle. Dauto (talk) 15:51, 27 July 2011 (UTC)[reply]
This report quotes Fermilab theorists speculating that there might be up to five kinds of "Higgs" bosons. -- Finlay McWalterTalk 15:55, 27 July 2011 (UTC)[reply]
That's what I was talking about except that they are counting particles differently than what I did. There is a pair of charged Higgses which are anti-particles of each other. If you count those separately, you have a total of five. If, on the other hand, you chose, as i did, to count them only once, then you get a total of four. Dauto (talk) 19:36, 27 July 2011 (UTC)[reply]
Well there was this idea once that there was only one electron and it bouncing back and forth in time made all the electrons and positrons in the world and that's why they all had the same charge. Perhaps that has made a comeback? ;-) Dmcq (talk) 17:42, 28 July 2011 (UTC)[reply]
One-electron universe -- Finlay McWalterTalk 17:49, 28 July 2011 (UTC)[reply]


July 28

Please identify this Dragonfly species

Please help identify this dragonfly species. The picture was taken in New Jersey on a concrete surface. The animal was just under three inches in length. μηδείς (talk) 04:49, 28 July 2011 (UTC)[reply]

My guess is this is a Common Whitetail, check the "adult female" in that article. --Jayron32 04:56, 28 July 2011 (UTC)[reply]
There are also several close matches at the genus Libellula, for example check the adult female example at Twelve-spotted Skimmer. --Jayron32 04:59, 28 July 2011 (UTC)[reply]

Funny, I told my informant she would get a quick response. Certainly looks like Libulella. I'll post her opinion after I call her tomorrow. Thanks. μηδείς (talk) 05:04, 28 July 2011 (UTC)[reply]

Thanks. She is satisfied it could be the female of either species. μηδείς (talk) 19:09, 28 July 2011 (UTC)[reply]
Resolved

Any research or speculation in medical literature on using radiation or chemotherapy to treat obesity?

Hi, I was wondering if cancer treatments like chemotherapy and radiation could be modified to treat obesity,(as well as infection). Has anyone seen anything written on this, pro or con, in the medical lit? -Thanks, Rich Peterson24.7.28.186 (talk) 06:02, 28 July 2011 (UTC)[reply]

I'm confused, are you sure you mean obesity? Obesity is not caused by any "agent" that could be targeted by chemo or radio therapy. How do you imagine those treatments could be modified to treat obesity? Vespine (talk) 06:12, 28 July 2011 (UTC)[reply]
It would be straightforward to treat obesity using chemotherapy or radiation: both of them tend to cause severe nausea, which predisposes against eating. The problem, of course, is that most people see the "cure" as worse than the disease. Looie496 (talk) 06:34, 28 July 2011 (UTC)[reply]
If the goal is simply to discourage appetite through prolonged, severe nausea, there are many other things one could consume that don't carry the risk of severe side effects (everything from permanent organ damage through to secondary cancers) associated with antineoplastic drugs or ionizing radiation. TenOfAllTrades(talk) 13:19, 28 July 2011 (UTC)[reply]
Sounds more like melting the fat away. You'd make billions. μηδείς (talk) 18:56, 28 July 2011 (UTC)[reply]
Well, thermal radiation (as well as thermal conduction and convection) is a rather old method of reducing weight, as in a sauna, but may cause weight loss mainly due to dehydration. StuRat (talk) 23:22, 29 July 2011 (UTC)[reply]

Horse manure

I saw this on the CBC web site:

But why is horse manure different? Why don't police have to stoop and scoop like everyone else? Sgt. Kristopher McCarthy, of the mounted unit, says that unlike dogs horse droppings have no harmful bacteria. "The difference between dogs and horses is that dogs eat meat and horses do not eat meat," McCarthy said.

Is it true that because horses are herbivorous, horse poop has “no harmful bacteria”? Follow-up question: what about humans who are vegan, does their poop have no harmful bacteria? Mathew5000 (talk) 12:19, 28 July 2011 (UTC)[reply]

No, harmful bacteria originate from external sources - they are deposited post deffication. The digestive tract is a pretty harsh environment for bacteria to survive, so only a select group of bacteria can do so. That being said, a carnivorous diet should provide a more nutritious bacteria culture. The point is that, the feces only becomes hazardous after defecation with the progression of time.I'd be more worried about deseases and other-nonbacterial organism that can accompany the feces. I am not a biologist, I am confident in my answer, nonetheless I welcome correction. Plasmic Physics (talk) 12:47, 28 July 2011 (UTC)[reply]
Seems like a case of basically right answer, but for the wrong reasons. This does ring true "Within two to three days manure will just dry out and blow away, very similar to clippings of grass." Also my wp:or indicates that carnivores do have worse-smelling feces than herbivores. SemanticMantis (talk) 14:02, 28 July 2011 (UTC)[reply]
The stomach is a harsh environment for most types of bacteria, but the lower parts of the digestive system are very hospitable. The digestive system of a horse in fact contains huge quantities of bacteria, and the horse depends on them to survive. In common with other ruminants, the only way they can digest cellulose is by allowing bacteria to ferment it in a structure called the cecum. The large intestine is also massively colonized by bacteria, as it is in all mammals. Looie496 (talk) 17:02, 28 July 2011 (UTC)[reply]
And even with all of that, horses pass 3/4 of their dinner undigested. --jpgordon::==( o ) 18:14, 28 July 2011 (UTC)[reply]
I used to have a neighbour who had a rather unpleasant method for dealing with dog feces on his backyard lawn. He would simply irrigate the lawn, feces and all, and then mow the lot. It's unpleasant due to the horrific smell drifting over the fence. Plasmic Physics (talk) 14:40, 28 July 2011 (UTC)[reply]
I can see why he's no longer your neighbor... Whoop whoop pull up Bitching Betty | Averted crashes 02:44, 31 July 2011 (UTC)[reply]
Horse manure can be fairly disgusting also, and the police are not unaware of that. I vaguely recall there was one protest in Chicago that they apparently must have disapproved of (maybe it was against the drug war) - think it was around the 96 DNC - which they insisted to "crowd control" on horseback. And they must have fed those horses every morsel they could cram down, because they (the horses that is) defecated continuously for the entire short route. The crowd thus had to keep on its toes, but I'd say the joke was on the police in the end, who ended up having to keep their horses in a neat little line for what seemed like hours as some speakers went on at great length ... all the while accumulating a reeking outdoor toilet behind them of positively epic proportions. Now I'm not entirely sure this was the reason, but Chicago eventually started requiring horse diapers in many cases. [3] Wnt (talk) 14:57, 28 July 2011 (UTC)[reply]
I guess you linked to that website about the diaper bit but it actually says horse urine smell is a bigger problem then the manure:
According to Sam, horse urine "smells like pure ammonia." He literally threw up one day because of the overwhelming odor, and he spent the rest of the summer with Vicks VapoRub under his nose, all because of the urine smell ("Shit actually doesn't smell that bad. I'd rather clean horse manure than people manure.").
Nil Einne (talk) 08:27, 29 July 2011 (UTC)[reply]
Keep in mind that the reason we have pooper-scooper laws is because of hysteria about parasites (hookworms, tapeworms and roundworms) being transmitted to babies playing with dog feces. It was never about bacteria. - Nunh-huh 21:17, 29 July 2011 (UTC)[reply]

Baking powder

Baking powder was not around during the colonial days. It was invented during the middle to late 1800's. What did the colonial women use to make cakes rise? If anyone can answer this, I would be grateful. — Preceding unsigned comment added by TerrDA (talkcontribs) 13:30, 28 July 2011 (UTC)[reply]

Baking powder#History Plasmic Physics (talk) 13:46, 28 July 2011 (UTC)[reply]
Baking powder is a leavening agent; that article lists several alternatives, including mechanical (whisking) leavening. The "history" section of the cake also talks about various pre-baking-powder cakes and cake-like desserts. It's also noteworthy that lots of cultures used, and use, unleavened foods, like pastries, fritters, or dumplings, in the role that you might expect cakes to take in modern western cuisine. -- Finlay McWalterTalk 13:52, 28 July 2011 (UTC)[reply]
Baker's Yeast has been used since at least the ancient Egypt, possibly much longer. Dauto (talk) 16:09, 28 July 2011 (UTC)[reply]

Baking soda predates baking powder, but probably the most common way of leavening a cake at that time was to use beaten egg whites -- that's still the method used for making sponge cakes. Looie496 (talk) 16:53, 28 July 2011 (UTC)[reply]

Leavening can happen with natural airborne yeasts as this recipe attests. You just need to leave the dough to stand for a few days. See also Sourdough; "Sourdough starter is made with a small amount of old dough saved from a prior batch, and is sometimes called mother dough or chef. This small amount of old-dough starter contains the culture, and its weight is increased by additions of new dough and mixing or kneading followed by rest or leavening periods. A small amount of the resulting dough is then saved to use as old-dough starter for the next batch. As long as this starter culture is fed flour and water weekly, it can stay at room temperature indefinitely." The article also says that cultured yeast followed the discoveries of Louis Pasteur; before that, Barm from brewing was used. Alansplodge (talk) 19:12, 29 July 2011 (UTC)[reply]
Then there's salt rising bread, intriguingly made using bacteria better known for food poisoning and gas gangrene, which are mysteriously docile when handled this way. Wnt (talk) 07:02, 30 July 2011 (UTC)[reply]

Lovely mystery mushroom

Does anybody recognize what this mushroom is? It was spotted while hiking Freeman Creek in the southern Sierra Nevada. --jpgordon::==( o ) 14:26, 28 July 2011 (UTC)[reply]

Given the location in the Sierras, Calvatia sculpta seems a good possibility. But I'm no expert. Deor (talk) 15:09, 28 July 2011 (UTC)[reply]
That looks vaguely reminiscent of the puffball Calvatia sculpta, from about the right area, not close enough to convince me. Wnt (talk) 15:15, 28 July 2011 (UTC) -- note I added this without seeing the previous comment. Interesting we both picked the same thing - in any case I don't mean to naysay the previous poster. Wnt (talk) 15:16, 28 July 2011 (UTC)[reply]
I'd venture Calvatia sculpta is correct. It was in the right elevation range in the right location. Cutting one in half to see the puffball insides would be a good way to tell, I suppose, though there doesn't seem to be much to confuse it with. (No, I'm not planning on either eating one or destroying one; they're too pretty on the forest floor.) Thanks! --jpgordon::==( o ) 15:25, 28 July 2011 (UTC)[reply]
"It was spotted while hiking" - now that is impressive. AndrewWTaylor (talk) 16:14, 28 July 2011 (UTC)[reply]
This is the Science Refdesk. Relativity applies here. Smile and the world smiles with you - hike and the world hikes against you. Wnt (talk) 18:04, 28 July 2011 (UTC)[reply]
If we could get a picture of that mushroom's backpack it might help to identify it. Bus stop (talk) 18:20, 28 July 2011 (UTC)[reply]
I wonder if he's ever shot a picture of a mushroom wearing his pajamas. :-) StuRat (talk) 23:13, 29 July 2011 (UTC) [reply]
Knowing the photographer, he has shared his pajamas, yes. --jpgordon::==( o ) 15:37, 30 July 2011 (UTC)[reply]

Dehumidifier in the desert

I was emptying out my dehumidifier when I had this idea: in places where open water is scarce, like in the Sahara region, why don't they use dehumidifier-like things to extract the water from the air? I understand that the air in those places is obvious not very humid, but even with 1% humidity, one cubic metre of air would yield 10 l of water, which is enough for one person to drink, and maybe even cook if used frugally. Solar power could be used to power the dehumidifier where electricity is expensive or unavailable. Has this been tried before? I'd like to read about it if it has. Thanks in avance. — Preceding unsigned comment added by 118.98.102.191 (talk) 18:16, 28 July 2011 (UTC)[reply]

I think you're high by several orders of magnitude: the water content would be measured, even at high RH, in terms of grams of water per kilogram of air. Acroterion (talk) 18:21, 28 July 2011 (UTC)[reply]
Yes, see our humidity article. Air at '100% humidity' isn't all water (obviously), but instead air with the maximum possible amount of water vapour present at that particular temperature and pressure - any surplus vapour would condense out as mist etc. AndyTheGrump (talk) 18:27, 28 July 2011 (UTC)[reply]
The BBC Weather page for Riyadh gives tomorrow's temperature as 44C with RH of 12%. Using this table (linked from the humidity article) shows that the air there will contain ~7g of water per cubic metre - about a teaspoonfull. -- Finlay McWalterTalk 18:35, 28 July 2011 (UTC)[reply]
But the amount of air which will go through the dehumidifier is far more than a single cubic meter. Whoop whoop pull up Bitching Betty | Averted crashes 02:40, 31 July 2011 (UTC)[reply]

Air well (condenser), Dune technology μηδείς (talk) 18:55, 28 July 2011 (UTC)[reply]

Here's a common method to capture water in the desert: dig a pit about one metre across and deep, then place an open empty water bottle in the centre of the pit. Spread a plastic tarp over the entire pit and fasten it using pins into the sand. Place a small rock in the centre of the tarp over the bottle, and the moisture will rise onto the tarp, condense and fall into the bottle. ~AH1 (discuss!) 23:24, 28 July 2011 (UTC)[reply]
One practical problem with using a dehumidifier or air conditioner to collect water in the desert is that it would immediately evaporate. For a given drop of water, you might only have seconds to get it into a sealed container before it would become vapor. This would especially be a problem on a small scale. StuRat (talk) 23:10, 29 July 2011 (UTC)[reply]
A dehumidifier is essentially a refrigerator - it only works if the air gets cold enough that dew would form naturally (below the dew point). Given that the water condenses in some refrigerated space, from refrigerated air, it shouldn't re-evaporate afterward. But since temperatures in deserts often vary greatly, it may be possible to use some apparatus that doesn't consume energy to accomplish this task. Wnt (talk) 07:01, 30 July 2011 (UTC)[reply]

Horses whinnying at bicyclist

More than once, while passing a horse going in the opposite direction on a country lane in the UK, the horse has whinnied. Has anyone else experienced this? Does this suggest that some horses think of bicycles plus rider as a kind of horse? 92.24.133.177 (talk) 18:44, 28 July 2011 (UTC)[reply]

Could also be fear of the sort Temple Grandin describes with startled cows, see Animals in Translation, as well as Monty Roberts. μηδείς (talk) 19:15, 28 July 2011 (UTC)[reply]
I've had horses respond to me as a cyclist producing Horse Noises. Nothing too unusual. ~AH1 (discuss!) 23:17, 28 July 2011 (UTC)[reply]
What do you mean by horse noises? 92.29.113.104 (talk) 10:19, 29 July 2011 (UTC)[reply]
Horse noises. μηδείς (talk) 20:20, 29 July 2011 (UTC)[reply]

Trans fat in British soft margarine

When I buy a tub of margarine in the UK, what are the chances of it containing more than trace amounts of trans fat? 92.24.133.177 (talk) 19:43, 28 July 2011 (UTC)[reply]

Here in Canada, any processed food containing less than 0.5 grams of trans fat per indicated serving amount can be advertised as "zero trans fat". Always check the nutrition facts. ~AH1 (discuss!) 23:16, 28 July 2011 (UTC)[reply]

Trans fat is never shown in the nutrition label in the UK. The manufacturers and government regulators keep us in the dark. 92.29.113.104 (talk) 10:14, 29 July 2011 (UTC)[reply]

In that case, the best you can do is look for hydrogenated vegetable oils in the ingredients list, unless a reliable lab has test results available, or the company's own webs site clarifies matters. StuRat (talk) 23:04, 29 July 2011 (UTC)[reply]
Wow. According to this some politicians in Britain say they're OK if they only make up "1% of total energy". But it also says that some members of the British Retail Association like Tesco have removed it from own-brand foods. Maybe that's a lead? I oppose nanny-state regulation but I do think something sold as food should be made out of food, not something chemically processed at high temperatures over a catalyst into substances not naturally encountered in the diet. Wnt (talk) 06:47, 30 July 2011 (UTC)[reply]
I'm opposed to a "nanny state", too, but, in order for people to be allowed to make their own decisions, they do need information, such as whether their food contains this poison, or not. StuRat (talk) 04:37, 31 July 2011 (UTC)[reply]
Hey y'all, remember the famous saying, "The dose makes the poison". Sure enough, in large quantities trans-fats are harmful, but there's no harm in consuming small amounts of trans-fat. This whole issue has been vastly overblown by the so-called "healthy food movement" (which IMHO is more concerned with opposing capitalism than with improving public health). Anyway, I often eat Oreo cookies and other stuff like that, and I've never experienced any ill-effects from it. Yes, more info might be in order here, but we must keep in mind that trans-fats are not a "poison", just an ingredient that's not healthy for you in large amounts (but perfectly OK in small amounts). As another saying goes, "Everything in moderation". 67.169.177.176 (talk) 05:13, 31 July 2011 (UTC)[reply]
Well, I wouldn't expect any obvious effects from trans-fats until your first heart attack. (Trans-fats actually cause me to get acne, but that's apparently just me.) StuRat (talk) 06:08, 31 July 2011 (UTC)[reply]
And if you keep trans-fat consumption to a low level, I bet you never will. 67.169.177.176 (talk) 08:38, 31 July 2011 (UTC)[reply]
Right, but this requires knowledge of which foods contain trans-fats, and in which quantities; hence this question. StuRat (talk) 18:33, 31 July 2011 (UTC)[reply]
Apart from health issues, IMHO eating trans fats makes me get an "old stopped-up sink taste" in the back of my throat. (Supposedly humans don't have taste/smell like receptors in the back of the throat like other mammals but I don't believe it) Wnt (talk) 20:37, 1 August 2011 (UTC)[reply]

Chocolate for treating depression

Does any study indeed treated depressed people with chocolate to test its anti-depressing effect? (kind of 100 gr. black chocolate in the morning). Quest09 (talk) 20:33, 28 July 2011 (UTC)[reply]

Yeah. It doesn't work so well. Please see PMID 16546266. 99.39.4.220 (talk) 21:08, 28 July 2011 (UTC)[reply]
A placebo might work, though depression is not purely physiological. ~AH1 (discuss!) 23:13, 28 July 2011 (UTC)[reply]
I don't know about an official study, but I think most married men would agree that it works on their wives... Beeblebrox (talk) 19:35, 30 July 2011 (UTC)[reply]
Very nice, Beebles. Whoop whoop pull up Bitching Betty | Averted crashes 02:49, 31 July 2011 (UTC)[reply]

Should I get a food processor, mixer, or something else?

My dad doesn't want to have his two unstable teeth pulled for a full denture, but he can't chew vegetables as a result. I'm thinking a food processor would make such an important food group more palatable. Do you agree? I don't think a mixer would be as appropriate? Imagine Reason (talk) 23:08, 28 July 2011 (UTC)[reply]

Maybe try a juicing machine for some vegetable juice, such as brocoli? ~AH1 (discuss!) 23:11, 28 July 2011 (UTC)[reply]
I wouldn't wish a juicer on anyone other than convicted war criminals. You feed the thing £10 of nice fruit and veg and it emits a pint or so of a murky fluid that tastes downright suspicious and causes alarming gastrological disturbances, together with a completely impossible volume of fluffy brown loft insulation. Then you have to spend half an hour cleaning weird paste from the little ducts and sluices in the machine, and another half hour removing tiny seeds from its spinny juicing disc. This explains why charity chops are full of shiny new juicers (on the same shelf as the bread-flavoured-goo-makers). -- Finlay McWalterTalk 23:32, 28 July 2011 (UTC)[reply]
I have an immersion blender which came with a food-processor add-on (so you can chop up veg to make things like soup or salsa). I mostly use it to make smoothies, and my mum uses hers to make creamed (that is, fully liquid) vegetable soups. Its best feature is that it's trivial to clean (one runs the immersion part under a running tap), which means you're happy to use it very frequently (a juicer, in contrast, is torment to clean). In my experience toddlers will happily drink veg (things like carrots) if it's hidden in a smoothie; mayhap your tooth-challenged eldster will feel similarly. -- Finlay McWalterTalk 23:22, 28 July 2011 (UTC)[reply]

Juicers give excellent results from carrots and grapes. Urrghl. But were I starving I'd prefer a blender. μηδείς (talk) 04:55, 29 July 2011 (UTC)[reply]

They work pretty well with apples too. 67.169.177.176 (talk) 05:16, 31 July 2011 (UTC)[reply]
How about using a potato masher on the cooked vegetables? 92.29.113.104 (talk) 10:16, 29 July 2011 (UTC)[reply]
This is kind of what I'm looking for. I don't believe in juicers, because they leave behind much of the nutrients. Grapes are high in fructose as well. Imagine Reason (talk) 14:49, 29 July 2011 (UTC)[reply]
Try this: [4]. ~AH1 (discuss!) 20:11, 29 July 2011 (UTC)[reply]
Actually, except for the fiber of a vegetable's cell walls, juicers do extract the large portion of the water soluble nutrients. μηδείς (talk) 20:17, 29 July 2011 (UTC)[reply]
Right, so it follows that for those people who are concerned that they are getting too much fiber in their diets, juicers are appropriate. The rest of us should avoid them. StuRat (talk) 23:01, 29 July 2011 (UTC)[reply]
I think the OPs father would more enjoy his vegetables chopped up or mashed up into small pieces that did not require chewing, rather than just making them into a soup as a blender or liguidiser would do. So I suggest either chopping then finely or using a potato masher as previous suggested. And few people get enough fibre, so it would be unhealthy to discard the fibre by using a juicer. 92.29.124.70 (talk) 12:28, 30 July 2011 (UTC)[reply]
  • There are many healthy, delicious things you can make with veggies in a food processor. Ajvar or Baba Ganoush can be used as a templates for further experimentation. By varying the base ingredients and spices the possibilities are endless. Another thing you can do is to sweat carrots, onions, celery, etc, then puree them in the processor and incorporate them into another dish. This is a great way to make various kinds of soups and bean dishes. If its hot out Gazpacho can be quite refreshing. Beeblebrox (talk) 19:45, 30 July 2011 (UTC)[reply]

July 29

Radioactive casting

When radioactive metals are molten down and cast into ingots, how is the critical mass taken into account when melting down the metal? How is the process different from handling non-radioactive metals? What shapes are typical for radioactive ingots? I have an idea, for a shape: hexagonal column with a hole through the middle, the diameter of the the hole being proportional to the fissile cross section. Plasmic Physics (talk) 03:46, 29 July 2011 (UTC)[reply]

I think nobody ever dared to produce an ingot larger than the critical mass of the fissible isotope. As a lot of radioactive elements are not undergo fission for most of the radioactive isotopes you do not have this problem.--Stone (talk) 08:23, 29 July 2011 (UTC)[reply]

Surely, more than one ingot's worth of metal is melted down at once? Plasmic Physics (talk) 11:08, 29 July 2011 (UTC)[reply]

Hollow spheres and columns are often used for storing very high grade fissile material for the very reason you've mentioned — the air gap increases the critical mass considerably. --Mr.98 (talk) 10:26, 29 July 2011 (UTC)[reply]

What is the safety margin, by how much percent is the ingot from critical mass? How about radioactive foil, possibly laminated between layers of wax, for the more radioactive metals? Plasmic Physics (talk) 11:08, 29 July 2011 (UTC)[reply]

The requirements are quite detailed and baroque. Calculating critical masses requires a lot of careful work based on the geometry, composition, and neutron moderation or reflection issues. I'm not sure there is one "safety margin" figure. The NRC has all sorts of regulations and approved containers and etc. and these have been designed and reviewed by great numbers of engineers.
Wax is probably not a good thing to use in a radioactive context, because it likely would slow neutrons down (acting as a moderator), which actually increases induced radioactivity and fissioning. --Mr.98 (talk) 11:40, 29 July 2011 (UTC)[reply]

How about a silicon based wax? If not, what would you use to protect the foil surface from oxidation, while simultaneously acting as a spacer, and must be flexible? Plasmic Physics (talk) 15:07, 29 July 2011 (UTC)[reply]

What your looking for are things like the "upper sub-critical limit" etc. This resent document goes into the Canadian regulations in some detail. Nuclear Criticality Safety Regulatory Document RD-327. This is not to say however, that following them to the letter will avoid you severely depressing your local real estate values and getting the neighbours grumpy. --Aspro (talk) 11:46, 29 July 2011 (UTC)[reply]

I was hoping for a percentage. For instance, the neutron flux must be no less than 40% from critcality at any point within the ingot. Plasmic Physics (talk) 15:07, 29 July 2011 (UTC)[reply]

Having a molten fissile elements with more than a critical mass just sounds like such a bad idea. There are fission reactors that use uranium metal alloys, but the most common fissile fuel is uranium oxide (UO2), which is produced as a powder (from U3O8 and UF6), packed into molds and sintered together in a furnace. The production process never requires molten uranium. For many of the same reasons that one would have to be careful about molten uranium, there are presumably very detailed rules for the handling and concentrations of such powder to avoid a criticality event; however, the powder has the advantage that it can be handled and formed at room temperature. Many of the alternatives of uranium oxide fuel, such as uranium nitride, uranium carbide, and plutonium oxide, are similarly formed via powders rather than as pure metals. Dragons flight (talk) 16:01, 29 July 2011 (UTC)[reply]

Post graduate study in the renowned medical colleges in U.S.

After we are done studying the under graduate classes for M.B.B.S. , we have to have the M.D. degree. So are there scholarship oppurtunities for deserving students to have post graduate study in the renowned medical colleges?113.199.183.176 (talk) 04:01, 29 July 2011 (UTC)[reply]

Try checking the websites of those specific colleges, or Google American medical scholarships and Nepalese medical scholarships for your country of origin, based on your IP address. ~AH1 (discuss!) 20:08, 29 July 2011 (UTC)[reply]

Brain in the heart

There have been several articles on the web spreading this rumor? Are there any reliable sources that talk about heart thinking and memorizing?--Almuhammedi (talk) 08:09, 29 July 2011 (UTC)[reply]

Sounds like nonsense. Anyway as for RS, the best I could find is [5] which appears to have been published [6]. Although for some reason on the Royal College of Psychiatrists website, glancing thru it and seeing discussions on things like the human 'spirit' and bioelectromagnetic communication between people doesn't exactly give me confidence in it, perhaps partially explaining why it was published in the The Arab Journal of Psychiatry Nil Einne (talk) 08:53, 29 July 2011 (UTC)[reply]
It doesn't take much googling to find people claiming that this discovery that our brains are "really" in our hearts is the literal fulfillment of a Qur'an passage. Maybe this is the Islamic equivalent of the "Lost Day" urban legend. APL (talk) 09:09, 29 July 2011 (UTC)[reply]
Seems to have root in this research here where it is apparently discovered that the heart has a small nerve cluster that is used to control and regulate the heart. Probably because the researcher seems to have dubbed this a "little brain", all the crazy pseudo-scientists have latched onto it and made up all manner of crazy stories.
However this nerve cluster is obviously not sophisticated enough to do anything the potions and crystals crowd says it does, if only for the fact that it's tiny. Ants have brains almost ten times larger. APL (talk) 09:02, 29 July 2011 (UTC)[reply]
The heart does have a system of cardiac ganglia (little clumps of neurons). Here's a diagram showing where all the cardiac ganglia are.[7] There are neurons connected to the heart for several reasons. Some of those neurons are a part of the parasympathetic nervous system, a system of neurons that regulates the body's functions when the body is at rest. Other of the heart's neurons are part of the sympathetic nervous system, which prepares the body for a fight-or-flight response by, for example, speeding up the heart. And still other of the heart's neurons are a part of the sensory system. But saying that the heart "has a brain" just because it has some neurons is really an exaggeration. The heart's neurons just deal with low-level bodily functioning. There really aren't enough neurons involved to call what they do "thinking". It just looks like some new age types have exaggerated, twisted, and adapted something that started off as a bit of scientific reality to meet their own beliefs and viewpoints, similar to how the flaky quantum mysticism grew out of the scientific quantum mechanics. Red Act (talk) 09:58, 29 July 2011 (UTC)[reply]
There's a little more to it than that. The heart has a sort of central controller called the sinoatrial node, which serves as pacemaker for the whole system. It is connected to other parts of the heart by very specialized muscle cells called Purkinje fibers, which function almost like nerves. But I wouldn't myself call this a "brain", and I don't see anything mystical about it. Looie496 (talk) 16:29, 29 July 2011 (UTC)[reply]
Think of how clever a gnat is, with such a tiny brain. We can't rule out that there is some thought in the heart and other ganglia. There is an operation, vagotomy, which affects the vagus nerve further away from the brain than the heart, which has subjective effects on a person. The enteric nervous system is also dubbed a "second brain". Vagus nerve stimulation is even said to have anti-depressive effects. I don't think it's unreasonable to suppose that all the little (and large) ganglia throughout the body are part, one way or another, of "thinking", in some sense of the term. Of course, none of this changes that the biggest lump of grey matter is in the head. Wnt (talk) 17:08, 29 July 2011 (UTC)[reply]
The brain, heart and lungs are codependant. That's about it. ~AH1 (discuss!) 20:04, 29 July 2011 (UTC)[reply]
"We can't rule out" coupled with a total lack of evidence and a total lack of connection to what we do understand (in this case about the necessity of sense perception for thought) is what is called "faith". This, however, is the Science reference desk. μηδείς (talk) 20:12, 29 July 2011 (UTC)[reply]
Absence of evidence is not evidence of absence. And I described evidence - vagus nerve stimulation as an antidepressant. Of course, the theory to explain that may or may not involve any sort of "thought" in the heart, but it provides a starting point for new hypotheses. Science tells us we should keep an open mind and not declare that all thought (defined how?) must be in the brain. Wnt (talk) 20:31, 29 July 2011 (UTC)[reply]
Also note that according to James–Lange theory the heart rate itself is an integral part of the process by which we feel fear. Wnt (talk) 20:40, 29 July 2011 (UTC)[reply]
"Absence of evidence is not evidence of absence" is very shaky way to advocate believing in any specific thing. Absence of evidence is also an absence of reason to believe. (excepting religious faith.)
Besides, let's not forget that many individuals have had their hearts removed entirely and replaced with either someone else's heart or even a machine. Do they normally report a change in personality or thought processes? I've never heard of such a thing. APL (talk) 21:07, 29 July 2011 (UTC)[reply]

Utter bollocks. Axl ¤ [Talk] 10:10, 30 July 2011 (UTC)[reply]

Thanks for interactive explanations. One more question relevant; what are the current (approx) records of those lifespans who survived with artificial heart? Had some suffered brain problems as stated in the webs? By the way; not only some Muslims exaggerate with these rumors but some others as well (perhaps creationists in general are looking after such hopes to support their belief).--Almuhammedi (talk) 19:33, 30 July 2011 (UTC)[reply]

Why do horses whinny?

What is the purpose or function of whinnying, and other horse noises? I know nothing about horses. 92.29.113.104 (talk) 10:23, 29 July 2011 (UTC)[reply]

See communication. Cheers. --Jayron32 12:29, 29 July 2011 (UTC)[reply]
"Our analysis of the acoustic structure of whinnies of 30 adult domestic horses (ten stallions, ten geldings, ten mares) revealed that some of the frequency and temporal parameters carried reliable information about the caller’s sex, body size and identity." from this 2009 article. Sean.hoyland - talk 12:43, 29 July 2011 (UTC)[reply]

Is whinnying different from neighing, or are they just two different names for the same thing? Are these the only noises horses make? 92.29.113.104 (talk) 17:30, 29 July 2011 (UTC)[reply]

I don't know if equestrians distinguish whinny from a neigh. For instance, I think whickering is considered fairly distinct from either previous term, but wiktionary [8] just defines it with 'neigh'. I do know that many people can distinguish several types of dog bark, e.g. some barks loosely communicate 'hello, let's play', while other indicate 'back off, or I will bite'. --The point is, there are many, many vocalizations that horses make, and they can communicate many different things. I think this is more important to recognize than what names we call these sounds. Probably the best way to learn is to spend more time around horses and equestrians ;) SemanticMantis (talk) 18:45, 29 July 2011 (UTC)[reply]

Are there any new studies explaining this phenomenon in details?--Almuhammedi (talk) 13:06, 29 July 2011 (UTC)[reply]

this list has articles dating from as early as 1969 and as recently as 2009. --Jayron32 13:13, 29 July 2011 (UTC)[reply]
In the hope of pulling in other readers, we should explain that this is the disputed? observation that "hot water freezes faster than cold". I should say that what confuses me about it is: if 35 degree C water freezes faster than 5 degree C water, what temperature is it before it freezes? Isn't it 5 degrees C itself sometime? Though the "convection" explanation from the article might explain this... Wnt (talk) 20:25, 29 July 2011 (UTC)[reply]
You really need to read the article. The claim is not "disputed", the effect is just not what you said it is, that "hot water freezes faster than cold" in all cases. If the effect were simply that "hot water freezes faster than cold" than it would be essentially wrong. What the Mpemba effect covers is a narrowly constructed set of conditions where warmer water would be said to freeze faster than colder water would. It isn't a universal effect, but rather a minor effect in a well-constructed scenario. If applied to its correct conditions, it works, and therefore isn't generally disputed. There is some dispute about how important the effect is in the field of thermodynamics, but it is a real, repeatable experiment, if you do it right. --Jayron32 23:27, 29 July 2011 (UTC)[reply]
"Evaporation" also addresses that. The 35C water will eventually cool to 5C, but by then there might be a lot less of it.
If you ask me, this part of the effect is almost certainly why the old trick of tossing a pan of hot water into the air on a cold day will make snow, but a pan of cold water often won't. The tiny droplets in the air have a very large surface-area to volume ratio. APL (talk) 04:41, 30 July 2011 (UTC)[reply]

Transmitter question

what is the full form of STT in STT 433 MHz Transmitter? — Preceding unsigned comment added by 42.104.43.155 (talk) 14:30, 29 July 2011 (UTC)[reply]

Not sure, is that its brand name? Is it the company that operates the transmitter? Looking at the wikipedia article STT my best guess would be Singapore Technologies Telemedia or possibly Suomen Tietotoimisto. --Jayron32 14:46, 29 July 2011 (UTC)[reply]
A Google search says that it means "Sunrom Technologies Transmitter" -- Sunrom also makes a corresponding receiver called the STR-433. Looie496 (talk) 16:23, 29 July 2011 (UTC)[reply]
Another possibility: PTT is very common (especially in old-fashioned handsets, or in HAM radio gear). "PTT" means "push to talk" or "press to transmit" - i.e., hold down a trigger-button while speaking to transmit. STT, on the other hand, can stand for "Speak to Transmit" - in other words, instead of a button, you just talk into the handset or microphone, and it automatically transmits. You're always potentially transmitting (though you probably have a squelch circuit that may or may not be tunable). This mode is also known as "speakerphone," "squelch," "vox", "always on," "hot mic," and so forth. Nimur (talk) 18:15, 29 July 2011 (UTC)[reply]
See this document. There's really no doubt that it means "Sunrom Technologies Transmitter". The device is a little $5 chip used to build remote controls. Looie496 (talk) 22:11, 29 July 2011 (UTC)[reply]

How did Dewar make his flask

I saw a PDF available for $35 from the American Chemical Society, but wasn't going to pay that much just to satiate my curiosity. I just want to know how James Dewar constructed his Dewar flasks in the 1890s. I saw on the first page of the ACS paper how the larger outer flask, that had a tube near the neck for evacuating, was cleanly cut using a wire running a higher current and then pouring cold water on it, after which the smaller flask was placed inside and the outer flask melted back together, and I can imagine that glassblowers melted some glass to seal the portion between the two necks at the top, but I'm wondering how he quickly sealed off the tube at the top after having pumped out all the air. The pressure difference between the inside of the vacuum section and the room in which you're working would have made it hard to just use hot tongs and expect the glass to seal the hole nice and cleanly without molten glass getting sucked in and in general just getting a mucky mess (or just blowing a hole in the neck while you're heating it and ruining your vacuum). 20.137.18.50 (talk) 14:48, 29 July 2011 (UTC)[reply]

It's pretty easy to heat a glass tube and get it to constrict and seal under vacuum rather than the glass getting sucked back into the tube. See for example this tutorial. DMacks (talk) 15:01, 29 July 2011 (UTC)[reply]
The key point is that glass being, well, a glass doesn't have a single sharp melting point where it goes from completely solid to completely liquid. Instead it has a range of temperatures where it gradually softens. If you apply just enough heat, you can get it soft enough to deform and fuse, but not so soft it loses all structural integrity. So if you melt a glass tube under vacuum, the center of the zone you are heating, being the softest, will collapse inwards and it will do so without deviation as both ends of the tube are under vacuum. The areas adjacent to the center of the zone will be less deformable, and will resist getting sucked inward. And since it's not completely liquid, the adjacent areas will support the more deformable center zone. -- 174.24.213.112 (talk) 15:48, 29 July 2011 (UTC)[reply]
(ec) The last page of the document you (IP 20.137.18.50) linked contains this paragraph (hope this helps): "If new flasks are used it is unnecessary to clean them with cleaning solution, but they should be washed with stannous chloride, rinsed and silvered. Rinsing and silvering should be done with a small quantity of liquid, by shaking and rotating, rather than filling the entire space between the walls. Two or three coats of silver are desirable. The silver is then washed out of the evacuating tube with cotton soaked with very dilute nitric acid, the entire flask rinsed several times with water, and the evacuating tube necked down at F for sealing off. Evacuation is carried out at about 400° in the electric furnace over a period of thirty-six hours, by means of a mercury vapor pump and liquid-air trap. Evacuation for a shorter period of time fails to outgas the silver completely and the flask deteriorates rather rapidly. Sealing off is done as soon after turning off the heat as possible. Our practice is to anneal two of the Dewar flasks at once (this requires the moving of one while hot from one ring stand to another already in the furnace) and later to evacuate them together." -- Scray (talk) 15:17, 29 July 2011 (UTC)[reply]
Thanks, Scray. I didn't see that part. I wouldn't have thought they'd evacuate that way. I thought they would just attach a tube and turn on a vacuum pump (which I assumed they had in the 1890s). I'm going to have to search 'mercury vapor pump' and 'liquid air trap' now so I can at least visualize what they're talking about. 20.137.18.50 (talk) 16:50, 29 July 2011 (UTC)[reply]
No problem - I realized that you couldn't have seen that page without paying the fee; I have full access (being an academic). I'm glad that helped. -- Scray (talk) 22:22, 29 July 2011 (UTC)[reply]
To me, "mercury vapor pump" says diffusion pump, but those weren't invented until 1915. --Carnildo (talk) 00:51, 30 July 2011 (UTC)[reply]
Dewar was a technician of absolute genius quality. He seems to have been the first human to have devised a vacuum flask, (decades before 1915) with glass surfaces coated with mercury, to achieve the greatest possible thermal isolation of extremely low temp substances. Edison (talk) 02:59, 30 July 2011 (UTC)[reply]
Possibly a Sprengel pump 75.41.110.200 (talk) 03:09, 30 July 2011 (UTC)[reply]

Palladium

Why is palladium able to store so much hydrogen? --134.10.113.198 (talk) 16:30, 29 July 2011 (UTC)[reply]

See Palladium#Hydrogen_storage and Palladium hydride for a general overview. This google search turns up a veritable shitload of good sources which explore the mechanisms behind hydrogen absorption onto Palladium. This article goes into some really good details of the mechanism. You do need to be a little careful in your research, as the hydrogen/palladium system was the basis for the whole cold fusion bullshit back in the day; but also be aware that the system also has its more commonplace uses, like in catalytic hydrogenation, either as finely palladium on carbon, or in Lindlar's catalyst --Jayron32 17:02, 29 July 2011 (UTC)[reply]

Nightmares and Nightterrors.

Exactly what is the purpose of nightmares and nightterrors. --86.45.162.217 (talk) 20:14, 29 July 2011 (UTC)[reply]

I assume you aren't falling into the error of attributing purpose to evolution, but merely employing a common though technically incorrect idiom? That aside, not every biological phenomenon is a beneficial, selected character; some are just by-products of other biological aspects of an organism - to draw a computing software analogy, they're a bug, not a feature. {The poster formerly known as 87.81.230.195} 90.201.110.58 (talk) 21:38, 29 July 2011 (UTC)[reply]
This is OR, but I have most commonly had nightmares when something is wrong, most often I'm either too hot or having trouble breathing efficiently because something is covering my head. In those cases it seems like one function of the nightmare is to cause awakening. Looie496 (talk) 22:05, 29 July 2011 (UTC)[reply]
Like regular dreams, one possibility is that they allow us to review things that happened previously (bad things, in this case) to decide on alternative courses of action, or to imagine future scenarios, and develop a plan of action, should they occur during waking hours. Note that "things which happened" don't have to have been real, they could have been watched in a movie or on TV or merely been heard about. StuRat (talk) 22:48, 29 July 2011 (UTC)[reply]
This is like asking "what is the purpose of schizophrenia?" Sometimes there is a cause that can be pointed to, but purpose? I don't think so. Beeblebrox (talk) 23:16, 29 July 2011 (UTC)[reply]

Couldn't he get prosthetic implants?

Look at Nick: http://www.youtube.com/watch?v=1zzTFrmEQh4&feature=autoplay&list=WLE9874D88A2291309&index=42&playnext=2

If I were in his situation, I'd move heaven & earth to get implants of prosthetic arms and legs and never tire of that pursuit until I have them installed.

Would it be possible to have four prostheses installed on him? What's stopping him for having them? --70.179.165.67 (talk) 20:42, 29 July 2011 (UTC)[reply]

I can't speak to individual cases, but with prosthetic limbs costing upwards of $100,000 US per limb, expense might be a factor. - Nunh-huh 21:58, 29 July 2011 (UTC)[reply]
Do they still make wooden legs? Seems to me that would be far cheaper and better than nothing if you couldn't afford a very expensive high-tech leg. Nyttend (talk) 01:12, 30 July 2011 (UTC)[reply]
It looks like his situation is very different from a war amputee with a pegleg. His arms are simply not there period, no stump, I don't even know if the shoulder blades are there. There's nothing to strap a prosthetic on to, no easy means to control it. It is true that certain neuromuscular interface technology might be able to accomplish this, but this seems like it has been remarkably slow to develop, considering the encouraging preliminary results that people reported. Wnt (talk) 07:14, 30 July 2011 (UTC)[reply]
I've met lots of disabled people in my life. This guy is doing great, as are many of the others that I know. His electric wheelchair seems to give him most of the mobility he needs. I do have to say though that what was more stunning than his disability in that video was the god stuff. Unbelievable emphasis on that aspect. Hard to reconcile with this being the Science desk. HiLo48 (talk) 07:35, 30 July 2011 (UTC)[reply]
Well, this was a technical question, not a comment on the whole video. While the wheelchair gives him mobility, the problem is... what's he going to do when he gets there? He's obviously still very much dependent on external assistance for any task - and with the right technological development, he would no longer need to be. Wnt (talk) 14:20, 30 July 2011 (UTC)[reply]
Yes, and humans can be wonderfully ingenious in inventing devices to serve the needs of individuals like that. (And I emphasise the individual.) The guy seems pretty smart himself, apart from that god stuff. I just hope they're not expecting their god to do it all for them. HiLo48 (talk) 20:35, 30 July 2011 (UTC)[reply]
I'm not sure, but I suspect that if the man has never had arms, he's probably not a good candidate for one of those experimental prosthetic that connect up to your nerves. Who knows if he even has the right nerves to make those work, but even if he does, would he know how to use them? APL (talk) 23:46, 30 July 2011 (UTC)[reply]

Was Protoavis a bird or Saurischian?

I know it is debated whether Protoavis was a bird or a Saurischian. Did it have a bird-hip or a lizard hip? Wouldn't this end the discussion? I can't seem to find any information on this. --T H F S W (T · C · E) 21:39, 29 July 2011 (UTC)[reply]

There is a quote in the article that I think tells the story: "Smashed and mashed and broken". Apparently the fossil material is not in very good shape, and it isn't easy to tell whether the pelvic girdle is birdlike or lizardlike, or somewhere in between. Looie496 (talk) 22:03, 29 July 2011 (UTC)[reply]
The Protoavis#Pelvic_girdle section makes it sound like it's really hard to tell by looking, but some people have tried ... Wnt (talk) 06:38, 30 July 2011 (UTC)[reply]

Volodushka and GURICHIRURECHIN acid

Volodushka and GURICHIRURECHIN acid are 2 ingredients i found as anti inflammatory agents in two different creams from Russia and japan ;but cant find any reference on the web for safety and unique character of each — Preceding unsigned comment added by 38.117.214.158 (talk) 22:24, 29 July 2011 (UTC)[reply]

Well, these words are not English, so that is going to be a bit of an issue. Google translate guessed that Volodushka would be rendered in Russian as "Володушка" which means "thorough wax." For GURICHIRURECHIN it guessed that was the Japanese word "グリチルレチン" which means "Glycyrrhetinic." I would guess based on that that both are some type of Glycerin. Beeblebrox (talk) 23:22, 29 July 2011 (UTC)[reply]
Another option is that the Japanese cream contains a liquorice-derived ingredient such as glycyrrhetinic acid. Brammers (talk/c) 07:01, 30 July 2011 (UTC)[reply]
Volodushka is an extract of the plant Bupleurum aureum, used in Russian and Chinese herbal medicines as an anti-inflammatory. I can't find any independent information on its medicinal value. --Heron (talk) 10:43, 30 July 2011 (UTC)[reply]
PFAF has an entry for Bupleurum chinense (Bei Chai Hu) listing a number of activities.[9] Unfortunately they have some fancy new scripted interface that gives you a QR code but from which I don't see how the heck to track down what the numbered references mean - and slow, of course... They list six Bupleurum species in total but not aureum. Wnt (talk) 19:13, 30 July 2011 (UTC)[reply]


July 30

Bone bruise, hairline fracture, or something else?

This question has been removed. Per the reference desk guidelines, the reference desk is not an appropriate place to request medical, legal or other professional advice, including any kind of medical diagnosis, prognosis, or treatment recommendations. For such advice, please see a qualified professional. If you don't believe this is such a request, please explain what you meant to ask, either here or on the Reference Desk's talk page.
This question has been removed. Per the reference desk guidelines, the reference desk is not an appropriate place to request medical, legal or other professional advice, including any kind of medical diagnosis or prognosis, or treatment recommendations. For such advice, please see a qualified professional. If you don't believe this is such a request, please explain what you meant to ask, either here or on the Reference Desk's talk page. --~~~~

Sorry. APL (talk) 00:08, 30 July 2011 (UTC)[reply]

Just to go on the record here, I very heavily contest the fact that my question has been marked as a request for medical advice. The situation I am inquiring about occurred so long ago that I can't specifically remember when (probably 1-3 years ago), and the question requested information completely separate from advice (I've been around long enough that people should know by now I would be smarter than to ask for medical advice on Wikipedia)...it could have been answered with a simple link or explanation to the differences in symptoms between two conditions. In the interest of not going on an edit war I won't revert the deletion again (WP:BRD, if you will), but I strongly disagree with the question being considered a request for medical advice. Ks0stm (TCG) 01:11, 30 July 2011 (UTC)[reply]
I'd argue on your behalf, except ... I didn't know how to answer that question anyway. If it were easy to tell what happened by the difference in symptoms, there wouldn't be so many doctors sending people for X-rays. Wnt (talk) 07:05, 30 July 2011 (UTC)[reply]

Cloud phenomenon

Okay, I cave. I've watched several different videos of this phenomenon, so I'm fairly certain it's real, but I have absolutely no idea what's happening here or here. It seems like every time it shows up the person is behind the cloud from the sun, with the sun just below the lip of the top of the cloud, and the cloud has a diffuse, icy pileus around the top. Anyone care to speculate on what's causing these strange light "bouncing" phenomena? -RunningOnBrains(talk) 00:58, 30 July 2011 (UTC)[reply]

I don't know, but if I had to guess: it looks like a source of bright light sweeps across a cirrus cloud (which may be the anvil of a cumulonimbus cloud). The bright light appears to shift, with different "fronts" of light moving across the cirrus cloud quite quickly. Assuming that the only source of bright light is the sun, and that the atmosphere above the cirrus cloud is pretty boring, the only possibility that I see is that the light is being reflected from below. As it seems pretty sunny out, it doesn't seem like it could be reflecting from a lake with seiches or something - rather, my guess is that it's being reflected at an angle by ice crystals, perhaps in the cirrus cloud, more likely in the cumulonimbus formation just below it. Presumably as the winds shift, or the temperature of the air changes, the light is reflected from a different point or refracted in a different direction, causing the rapidly changing lights.
There's a theorem in astronomy that no body can change brightness in a faster time than it takes light to cross it. I wonder if something similar applies to a situation like this involving the speed of sound. Since the patterns seem to change within 0.5 to 1 seconds, I would picture that the reflecting feature should be less than about 150 to 300 meters in size. But I don't know if that's really valid. Wnt (talk) 06:34, 30 July 2011 (UTC)[reply]
As Wnt suggests,its a form of Sun dog phenomena. The moving clouds are repeatedly occluding a patch of ice crystals in the higher parts. --Aspro (talk) 09:39, 30 July 2011 (UTC)[reply]

Bone tracheas

I was quite confused by the following passage:

One skeleton taken out of this part of the mound had the appearance of a very aged man; the point of the inferior maxillary was almost in two parts, while the trachea was bone all around. Quite a number showed indications of extreme age; seven or eight that I observed had bone tracheas.

The context is a railroad official overseeing the excavation of a Glacial Kame archaeological site in the 1850s. So what's a bone trachea? I put this into Google and didn't find anything useful; most of the results were lists of body parts that happened to have "trachea" immediately following "bone". Nyttend (talk) 01:15, 30 July 2011 (UTC)[reply]
Cartilage tends to turn to bone over time. While some structures normally remain cartilaginous for life, ossification of tracheal cartilage can be a sign of advanced age. Ossification of tracheal cartilage in aged humans μηδείς (talk) 01:35, 30 July 2011 (UTC)[reply]
Try "trachea ossification" on google. Dauto (talk) 01:45, 30 July 2011 (UTC)[reply]
You will come up with the same paper I did above when I googled the same words. μηδείς (talk) 01:52, 30 July 2011 (UTC)[reply]
I always assumed that cartilage in childhood would normally remain cartilage throughout life, aside from issues such as arthritis; thanks for the help. I've now cited the linked PubMed article in a new article here about the site that was being excavated, the Ridgeway Site. Nyttend (talk) 02:33, 30 July 2011 (UTC)[reply]

The history and use of coal balls

The title says it all, pretty much. Does anyone know about the history of coal balls, and what they can be used for? --Σ talkcontribs 01:36, 30 July 2011 (UTC)[reply]

The only reference I can find to anything called "coal balls" at Wikipedia is found at the disambiguation page Niggerhead which indicates they were deposits of Pyrite found in coal mines. I have no idea if this is the item you are looking for. --Jayron32 03:43, 30 July 2011 (UTC)[reply]
I did find another reference to them, at Permineralization they seem to be small carbonate balls that represent the fossilized remains of microscopic plants. --Jayron32 03:51, 30 July 2011 (UTC)[reply]
A third possibility is the black fungus known as Daldinia concentrica. --Jayron32 03:53, 30 July 2011 (UTC)[reply]
I need information about the second possibility, about fossilised remains of plants. Thanks. --Σ talkcontribs 04:03, 30 July 2011 (UTC)[reply]
How about the mixture of clay and coal dust which Marco Polo wrote of and is used as a fuel in China today?—eric 02:59, 31 July 2011 (UTC)[reply]
Clay + coal dust is called a pencil. --Jayron32 04:53, 31 July 2011 (UTC)[reply]

Sounds of Earth from the Voyager Golden Record: volcanoes, earthquake, whale song, tractor... ?

I came across this article: Contents of the Voyager Golden Record, and I found myself trying to recognize the different sounds listed in the "Sounds" section. There is also a list on the NASA website: http://voyager.jpl.nasa.gov/spacecraft/sounds.html You can hear these sounds here: http://www.youtube.com/watch?v=s-6CvmmcG0w

I have several questions:

  • Does the sound of volcanoes really start at 00:43?
  • Is it the sound of an earthquake we can hear around 01:05?
  • The list on the NASA website doesn't mention "Whale song"; but is it that sound that starts at 04:57?
  • According to the list, there are two sounds of tractor, one that starts at 07:32 and one another at 09:25. The first one doesn't really sound like a tractor to me, rather like a jackhammer. What do you think?
  • The list mentions "Thunder" before "Mud pots" and "Wind" just thereafter, but I can't hear them. Is it an error? You can hear the sound of the wind at 05:27.

Also, would it be a good idea to rearrange the list and to add the timing, like in the "Greetings" section? Thank you! --Glups (talk) 04:05, 30 July 2011 (UTC)[reply]

The 7:32 tractor sounds like it has Caterpiller treads and no muffler. Edison (talk) 20:01, 30 July 2011 (UTC)[reply]

orbit and mass

Is the mass of a planet related in any way to its orbit? --DeeperQA (talk) 08:17, 30 July 2011 (UTC)[reply]

Absolutely. Please see our article on orbit: "Newton showed that, for a pair of bodies, the orbits' sizes are in inverse proportion to their masses, and that the bodies revolve about their common center of mass. Where one body is much more massive than the other, it is a convenient approximation to take the center of mass as coinciding with the center of the more massive body."--Shantavira|feed me 12:17, 30 July 2011 (UTC)[reply]
How does that conflict with the fact that massive planets - Mars, Jupiter - are in the middle tier of the Solar System, but small planets are on its extremes? — Preceding unsigned comment added by

88.8.79.148 (talk) 13:16, 30 July 2011 (UTC)[reply]

Mars is not particularly massive. Dauto (talk) 14:05, 30 July 2011 (UTC)[reply]
You are right on that. I was thinking about Saturn. 88.8.79.148 (talk) 14:20, 30 July 2011 (UTC)[reply]
No Shantavira, that's completely wrong. Newton has not showed any such relationship which is not observed in practice either. Our specific solar system's history has lead to some patterns described by Csmiller bellow but exoplanets have been found that do not follow that pattern. Dauto (talk) 14:05, 30 July 2011 (UTC)[reply]
Huh? I am quoting our article. Maybe you could fix it if it's wrong.--Shantavira|feed me 15:55, 30 July 2011 (UTC)[reply]
The quotation is correct. Your interpretation of it is wrong. That quotation simply mean that the size of the orbit of the moon around the earth is much bigger than the size of the orbit of the earth around the moon because the earth's mass is much bigger than the moon's mass. It does not mean that if the earth had a second moon, than the size of the orbit of that moon would have to correlate in any way with its mass. Dauto (talk) 18:54, 30 July 2011 (UTC)[reply]
See Formation and evolution of the Solar System#Formation of planets. Basically for Mercury to Mars, the Sun is hot enough to boil off most of the hydrogen from the planet. Further out the surface of the planetary core (which would been rocky or metally) would be cold enough for the captured hydrogen (and helium) to be captured - the thermal velocity of the gas is less than the surface escape velocity. In the trans-Neptunian area, there is not much hydrogen to be captured. CS Miller (talk) 13:30, 30 July 2011 (UTC)[reply]

It's not well understood at this time. Count Iblis (talk) 15:00, 30 July 2011 (UTC)[reply]

There are many satellites at altitude 35,786 km above the equator that have exactly 24 hours orbital period, see the article Geostationary orbit. Geostationary orbit is independent of the mass of the satellite. The orbital periods of planets about the Sun are similarly independent of their masses. Wikipedia has a long List of satellites in geosynchronous orbit that have a wide range of different masses. Cuddlyable3 (talk) 00:28, 31 July 2011 (UTC)[reply]

Does anyone know what this plant is, please?

A plant grew in my garden, and I've no idea what it is. Or was, as it grew on a long stem and looked temptingly like a cat toy, so my kitten attacked it, and it is no more. Here are a couple of photos: The entire plant: http://www.flickr.com/photos/63456161@N07/5990829659/in/photostream Close up of the head: http://www.flickr.com/photos/63456161@N07/5990829243/in/photostream/ Thanks for any information!Snorgle (talk) 19:23, 30 July 2011 (UTC)[reply]

Allium canadense --Digrpat (talk) 20:11, 30 July 2011 (UTC)[reply]
Thanks - that looks right(although I'm in the UK I suppose it could have been introduced), and it did smell garlicky too. The wiki entry suggests that it's edible AND poisonous, which is confusing, though.Snorgle (talk) 20:26, 30 July 2011 (UTC)[reply]
maybe Allium vineale my 1st guess anyway--Digrpat (talk) 20:33, 30 July 2011 (UTC)[reply]
That looks EXACTLY like it, thanks!Snorgle (talk) 23:24, 30 July 2011 (UTC)[reply]

Funny Science Video Clips

Can anyone suggest funny science clips like the one here Thanks! Barbaricslav (talk) 20:23, 30 July 2011 (UTC)[reply]

Turboencabulator. Cuddlyable3 (talk) 00:02, 31 July 2011 (UTC)[reply]
Unintentionally funny, but hilarious to me. -RunningOnBrains(talk) 01:03, 31 July 2011 (UTC)[reply]
The "Duck and Cover" routine is probably not as useless as a lot of people seem to think. Obviously if you're within a certain radius of the bomb, you're going to die regardless. The point of "duck and cover" is to save people who are further away from the blast.--Srleffler (talk) 04:39, 31 July 2011 (UTC)[reply]
I think the funny part is using a cartoon with a turtle and a jingle, instead of a serious discussion of thermonuclear war. I picture a series of these videos, where being raped and murdered is treated to a cartoon and jingle, then maybe having your school bus catch fire as everyone inside slowly burns to death. StuRat (talk) 04:44, 31 July 2011 (UTC)[reply]

July 31

Cold fusion

Cold fusion is referred to as "bullshit" above, and the article seems to say as much. I thought that there were reputable scientists, like those at the the San Diego Navy SPAWAR lab, who continue to produce confirmatory results and have never wavered in their support for cold fusion (e.g. [10]) and some Italian outfit that claimed to have built commercial scale reactors in the past year.[11] What is the current status of that controversy? 99.39.4.220 (talk) 00:54, 31 July 2011 (UTC)[reply]

The 'Italian outfit' can be found here: Energy Catalyzer. Opinions on its validity are, shall we say, divided. AndyTheGrump (talk) 00:58, 31 July 2011 (UTC)[reply]
(edit conflict)Certainly not bullshit. Nothing approached with an open mind and a receptive scientific attitude should ever be labeled "bullshit". There is certainly no convincing evidence that it exists (nor known theoretical mechanism by which it COULD exist). But people trying weird science is how some of the greatest advances of mankind have been achieved. Anyone claiming to have "proof" of anything does not deserve your attention. There is only evidence in science, never "proof". And until there is more than occasional irreproduceable evidence of the phenomenon, I say that there is no reason to believe it exists.
That said, the SPAWAR results are intriguing, but I think that we need to think about horses before zebras, in that there is likely some other energy source for the miniscule amount of heat they are able to produce. And I can not find anything about an Italian commercial application, but I can tell you that scam artists have been selling cold fusion kits for years.-RunningOnBrains(talk) 01:01, 31 July 2011 (UTC)[reply]
It is not bullshit to explore weird stuff in a scientific manner. Cold fusion should be researched as with anything else. It is bullshit to make definitive claims on the success of research when there are none. Let me reiterate: research = good, making up things = bullshit. Much of the problems with the cold fusion was too much bullshit, not enough good. --Jayron32 01:45, 31 July 2011 (UTC)[reply]
What he said ::points at Jayron:: -RunningOnBrains(talk) 02:27, 31 July 2011 (UTC)[reply]

Strength of atomic bonds

Hello... I was trying to get an idea of the strength of bonds between atoms by imagining that a water molecule, for example, was blown up to a size where I could hold it in my hands and pull it apart. However, I'm not sure how I should scale the force. Should it scale with the cube of the molecule length? Anyway, doing the calculation properly, what would be the force required to pull apart a water molecule if it was, let's say, 10 cm long? What would be the force required to separate two water molecules of that size in liquid water (keeping the individual molecules intact)? (Ignore the question of whether you could actually grasp such a blown-up molecule, and other similar quibbles.) 86.179.2.163 (talk) 01:20, 31 July 2011 (UTC)[reply]

I can't think of a single principle that would allow you to chose the proper scaling factor and in the absence of a principle your question is not answerable. Dauto (talk) 01:45, 31 July 2011 (UTC)[reply]
(edit conflict)Well, unfortunately, I do have a quibble. If you scale the size you still have to figure out how to scale all the other properties, like mass and charge, and this may not be a trivial answer. The H-O bond strength is 460 Joules...note that this is an energy required to break the bond, not a force. This is because at the molecular level the force needed to break a bond is unimportant, it is the energy input needed to break the bond that is constant. For this reason the was that you scale mass and charge becomes important. I feel like someone else should be able to explain this better, but suffice to say it is not as simple an analogy to scale up bond-breaking as it might seem.-RunningOnBrains(talk) 01:48, 31 July 2011 (UTC)[reply]
460 joules seems vastly too great a number? Did you miss off a factor of ten to the power minus something? Despite the objections, and despite not being able to work it out myself, I feel it ought to be possible to give a sensible answer. To put it another way, if I was shrunk down to the point where I could hold a water molecule in my hand, how hard would it be to pull it apart. It seems like a question that ought to have a sensible answer.... ! 86.179.2.163 (talk) —Preceding undated comment added 01:56, 31 July 2011 (UTC).[reply]
Actually, I was off in two ways. I was trying to say 460 kilojoules (110 kCal), but that's per mole, so take that number and divide by Avogadro's number to get the energy per bond. Sorry. Anyway, it is still a unit of energy, which is the crux of my argument. -RunningOnBrains(talk) 02:19, 31 July 2011 (UTC)[reply]
OK, well, how would the force generated by my muscles decrease as I shrank? If I could pull 100 kg at full size, and then I was halved in size, could I pull 100/8 kg? 86.179.2.163 (talk) 02:07, 31 July 2011 (UTC)[reply]
Do an experiment. Whoop whoop pull up Bitching Betty | Averted crashes 02:37, 31 July 2011 (UTC)[reply]
Silliness aside, your reasoning sounds correct. Various internet sources seem to claim that strength is connected proportional to muscle A) mass, B) volume, or C) cross-sectional area. Not sure which is correct, as no reliable source seems to weigh in on the matter as far as I can tell. -RunningOnBrains(talk) 02:47, 31 July 2011 (UTC)[reply]

Severed hands

If one's hand is sliced off, is he likely to bleed to death? Whoop whoop pull up Bitching Betty | Averted crashes 02:46, 31 July 2011 (UTC)[reply]

Why not? Cutting you wrists seem to be an effective mode of suicide. Plasmic Physics (talk) 03:07, 31 July 2011 (UTC)[reply]
Unreliable sources have claimed otherwise. Also i remember reading a statement (admittedly online) about cutting wrists hardly ever resulting in anything more than a scar (paraphrased near quote). Whoop whoop pull up Bitching Betty | Averted crashes 03:11, 31 July 2011 (UTC)[reply]
Cutting ones wrists often does result in a scar for a few reasons. Someone might find the person, call the local emergency services, and save the person. The person may just be trying to get attention and do it in front of someone who then calls. The person may not do it well enough, pass out, it clots, and they're found. Often the person doing the cutting doesn't do it well enough to actually exsanguinate themselves, passes out, wakes up later, cries, and goes on with their life. What do you expect, they're amateurs and probably don't know that much about what they're actually attempting. Dismas|(talk) 03:37, 31 July 2011 (UTC)[reply]
So, considering all that, what would completely severing the hand do? Whoop whoop pull up Bitching Betty | Averted crashes 03:57, 31 July 2011 (UTC)[reply]
If no medical attention were given, and no attempt was made to staunch the flow of blood? Kill them. If drastic lifesaving measures were used, they may survive. --Jayron32 04:22, 31 July 2011 (UTC)[reply]
Agreed. The most immediate difference would be made by applying a tourniquet. StuRat (talk) 04:23, 31 July 2011 (UTC)[reply]
Really? What to Expect the Toddler Years claims that applying pressure to the wound is usually sufficient to staunch the flow of blood even in such extreme circumstances. Whoop whoop pull up Bitching Betty | Averted crashes 04:32, 31 July 2011 (UTC)[reply]
Survival probably wouldn't depend on "drastic lifesaving measures". Some basic first aid would suffice. You're probably right, though, that left untreated it would be fatal.--Srleffler (talk) 04:47, 31 July 2011 (UTC)[reply]

IR tech

Can an infrared or thermal imaging sensor detect a person buried under snow? 67.169.177.176 (talk) 03:59, 31 July 2011 (UTC)[reply]

It depends on the density of the snow and the sensitivity of the sensor. Whoop whoop pull up Bitching Betty | Averted crashes 04:01, 31 July 2011 (UTC)[reply]
So a top-of-the-line IR sensor like those used by the military could do it easily, while a cheap $200 IR camera would be useless? Makes sense to me. BTW, am I right that you have an interest in aviation? 67.169.177.176 (talk) 04:11, 31 July 2011 (UTC)[reply]
Yup. Whoop whoop pull up Bitching Betty | Averted crashes 04:30, 31 July 2011 (UTC)[reply]
It would also depend on the depth of the snow and how long it has been in place. Right after an avalanche, the surface of the snow should be about the same temp everywhere, while, after some time, the snow near the body should be slightly warmer. StuRat (talk) 04:19, 31 July 2011 (UTC)[reply]
Unfortunately, by that time most of the victims would be dead. See Avalanche#Human survival and avalanche rescue. 67.169.177.176 (talk) 05:19, 31 July 2011 (UTC)[reply]

fart. Erm, flatulence

Exactly what is a fart composed of? Oh, that's obvious, it depends on what you eat/drink/do/say/whatever, but w What is the general base for a fart? Is it true that one can fart next to a lighter and get the fart fire cloud thingy? An editor since 10.28.2010. 05:37, 31 July 2011 (UTC)[reply]